Help in solving a problem posed in International Mathematical Olympiad 1983












0












$begingroup$


There was this question posed in IMO 1983:



"Can you choose 1983 pairwise distinct positive integers less than $10^5$, such that no three are in Arithmetic Progression?"



A close look at the sequence reveals the following things as mentioned in Problem Solving Strategies by Arthur Engel.




  1. When the numbers which belong to the sequence are written in the ternary system, all the digits in any of the number is either 0 or 1


  2. A further look suggests that the $nth$ term of the sequence has the same representation in the ternary system as $n$ has in the binary system



These help me to solve the original problem However, I am unsure as to how to prove the above two points



Any help please!!










share|cite|improve this question









$endgroup$












  • $begingroup$
    You don't have to prove they must be the numbers. You just have to prove those numbers work - that no three are in arithmetic progression.
    $endgroup$
    – Empy2
    Dec 13 '18 at 10:08










  • $begingroup$
    Can you help me to prove that please
    $endgroup$
    – saisanjeev
    Dec 13 '18 at 10:34










  • $begingroup$
    Given a,b in the sequence, prove the third term in arithmetic sequence must contain a 2 in ternary.
    $endgroup$
    – Empy2
    Dec 13 '18 at 10:40










  • $begingroup$
    ok. I tried it like this. Take a string of 1's and 0's in base 3, I multiply it by 2 to give a string of 2's and 0's with the 1's replaced by 2's. Now how do I prove that no matter what I subtract from the new number, I will always end up with a two in my answer?
    $endgroup$
    – saisanjeev
    Dec 13 '18 at 10:54
















0












$begingroup$


There was this question posed in IMO 1983:



"Can you choose 1983 pairwise distinct positive integers less than $10^5$, such that no three are in Arithmetic Progression?"



A close look at the sequence reveals the following things as mentioned in Problem Solving Strategies by Arthur Engel.




  1. When the numbers which belong to the sequence are written in the ternary system, all the digits in any of the number is either 0 or 1


  2. A further look suggests that the $nth$ term of the sequence has the same representation in the ternary system as $n$ has in the binary system



These help me to solve the original problem However, I am unsure as to how to prove the above two points



Any help please!!










share|cite|improve this question









$endgroup$












  • $begingroup$
    You don't have to prove they must be the numbers. You just have to prove those numbers work - that no three are in arithmetic progression.
    $endgroup$
    – Empy2
    Dec 13 '18 at 10:08










  • $begingroup$
    Can you help me to prove that please
    $endgroup$
    – saisanjeev
    Dec 13 '18 at 10:34










  • $begingroup$
    Given a,b in the sequence, prove the third term in arithmetic sequence must contain a 2 in ternary.
    $endgroup$
    – Empy2
    Dec 13 '18 at 10:40










  • $begingroup$
    ok. I tried it like this. Take a string of 1's and 0's in base 3, I multiply it by 2 to give a string of 2's and 0's with the 1's replaced by 2's. Now how do I prove that no matter what I subtract from the new number, I will always end up with a two in my answer?
    $endgroup$
    – saisanjeev
    Dec 13 '18 at 10:54














0












0








0





$begingroup$


There was this question posed in IMO 1983:



"Can you choose 1983 pairwise distinct positive integers less than $10^5$, such that no three are in Arithmetic Progression?"



A close look at the sequence reveals the following things as mentioned in Problem Solving Strategies by Arthur Engel.




  1. When the numbers which belong to the sequence are written in the ternary system, all the digits in any of the number is either 0 or 1


  2. A further look suggests that the $nth$ term of the sequence has the same representation in the ternary system as $n$ has in the binary system



These help me to solve the original problem However, I am unsure as to how to prove the above two points



Any help please!!










share|cite|improve this question









$endgroup$




There was this question posed in IMO 1983:



"Can you choose 1983 pairwise distinct positive integers less than $10^5$, such that no three are in Arithmetic Progression?"



A close look at the sequence reveals the following things as mentioned in Problem Solving Strategies by Arthur Engel.




  1. When the numbers which belong to the sequence are written in the ternary system, all the digits in any of the number is either 0 or 1


  2. A further look suggests that the $nth$ term of the sequence has the same representation in the ternary system as $n$ has in the binary system



These help me to solve the original problem However, I am unsure as to how to prove the above two points



Any help please!!







sequences-and-series elementary-number-theory






share|cite|improve this question













share|cite|improve this question











share|cite|improve this question




share|cite|improve this question










asked Dec 13 '18 at 10:04









saisanjeevsaisanjeev

1,073312




1,073312












  • $begingroup$
    You don't have to prove they must be the numbers. You just have to prove those numbers work - that no three are in arithmetic progression.
    $endgroup$
    – Empy2
    Dec 13 '18 at 10:08










  • $begingroup$
    Can you help me to prove that please
    $endgroup$
    – saisanjeev
    Dec 13 '18 at 10:34










  • $begingroup$
    Given a,b in the sequence, prove the third term in arithmetic sequence must contain a 2 in ternary.
    $endgroup$
    – Empy2
    Dec 13 '18 at 10:40










  • $begingroup$
    ok. I tried it like this. Take a string of 1's and 0's in base 3, I multiply it by 2 to give a string of 2's and 0's with the 1's replaced by 2's. Now how do I prove that no matter what I subtract from the new number, I will always end up with a two in my answer?
    $endgroup$
    – saisanjeev
    Dec 13 '18 at 10:54


















  • $begingroup$
    You don't have to prove they must be the numbers. You just have to prove those numbers work - that no three are in arithmetic progression.
    $endgroup$
    – Empy2
    Dec 13 '18 at 10:08










  • $begingroup$
    Can you help me to prove that please
    $endgroup$
    – saisanjeev
    Dec 13 '18 at 10:34










  • $begingroup$
    Given a,b in the sequence, prove the third term in arithmetic sequence must contain a 2 in ternary.
    $endgroup$
    – Empy2
    Dec 13 '18 at 10:40










  • $begingroup$
    ok. I tried it like this. Take a string of 1's and 0's in base 3, I multiply it by 2 to give a string of 2's and 0's with the 1's replaced by 2's. Now how do I prove that no matter what I subtract from the new number, I will always end up with a two in my answer?
    $endgroup$
    – saisanjeev
    Dec 13 '18 at 10:54
















$begingroup$
You don't have to prove they must be the numbers. You just have to prove those numbers work - that no three are in arithmetic progression.
$endgroup$
– Empy2
Dec 13 '18 at 10:08




$begingroup$
You don't have to prove they must be the numbers. You just have to prove those numbers work - that no three are in arithmetic progression.
$endgroup$
– Empy2
Dec 13 '18 at 10:08












$begingroup$
Can you help me to prove that please
$endgroup$
– saisanjeev
Dec 13 '18 at 10:34




$begingroup$
Can you help me to prove that please
$endgroup$
– saisanjeev
Dec 13 '18 at 10:34












$begingroup$
Given a,b in the sequence, prove the third term in arithmetic sequence must contain a 2 in ternary.
$endgroup$
– Empy2
Dec 13 '18 at 10:40




$begingroup$
Given a,b in the sequence, prove the third term in arithmetic sequence must contain a 2 in ternary.
$endgroup$
– Empy2
Dec 13 '18 at 10:40












$begingroup$
ok. I tried it like this. Take a string of 1's and 0's in base 3, I multiply it by 2 to give a string of 2's and 0's with the 1's replaced by 2's. Now how do I prove that no matter what I subtract from the new number, I will always end up with a two in my answer?
$endgroup$
– saisanjeev
Dec 13 '18 at 10:54




$begingroup$
ok. I tried it like this. Take a string of 1's and 0's in base 3, I multiply it by 2 to give a string of 2's and 0's with the 1's replaced by 2's. Now how do I prove that no matter what I subtract from the new number, I will always end up with a two in my answer?
$endgroup$
– saisanjeev
Dec 13 '18 at 10:54










0






active

oldest

votes












Your Answer





StackExchange.ifUsing("editor", function () {
return StackExchange.using("mathjaxEditing", function () {
StackExchange.MarkdownEditor.creationCallbacks.add(function (editor, postfix) {
StackExchange.mathjaxEditing.prepareWmdForMathJax(editor, postfix, [["$", "$"], ["\\(","\\)"]]);
});
});
}, "mathjax-editing");

StackExchange.ready(function() {
var channelOptions = {
tags: "".split(" "),
id: "69"
};
initTagRenderer("".split(" "), "".split(" "), channelOptions);

StackExchange.using("externalEditor", function() {
// Have to fire editor after snippets, if snippets enabled
if (StackExchange.settings.snippets.snippetsEnabled) {
StackExchange.using("snippets", function() {
createEditor();
});
}
else {
createEditor();
}
});

function createEditor() {
StackExchange.prepareEditor({
heartbeatType: 'answer',
autoActivateHeartbeat: false,
convertImagesToLinks: true,
noModals: true,
showLowRepImageUploadWarning: true,
reputationToPostImages: 10,
bindNavPrevention: true,
postfix: "",
imageUploader: {
brandingHtml: "Powered by u003ca class="icon-imgur-white" href="https://imgur.com/"u003eu003c/au003e",
contentPolicyHtml: "User contributions licensed under u003ca href="https://creativecommons.org/licenses/by-sa/3.0/"u003ecc by-sa 3.0 with attribution requiredu003c/au003e u003ca href="https://stackoverflow.com/legal/content-policy"u003e(content policy)u003c/au003e",
allowUrls: true
},
noCode: true, onDemand: true,
discardSelector: ".discard-answer"
,immediatelyShowMarkdownHelp:true
});


}
});














draft saved

draft discarded


















StackExchange.ready(
function () {
StackExchange.openid.initPostLogin('.new-post-login', 'https%3a%2f%2fmath.stackexchange.com%2fquestions%2f3037830%2fhelp-in-solving-a-problem-posed-in-international-mathematical-olympiad-1983%23new-answer', 'question_page');
}
);

Post as a guest















Required, but never shown

























0






active

oldest

votes








0






active

oldest

votes









active

oldest

votes






active

oldest

votes
















draft saved

draft discarded




















































Thanks for contributing an answer to Mathematics Stack Exchange!


  • Please be sure to answer the question. Provide details and share your research!

But avoid



  • Asking for help, clarification, or responding to other answers.

  • Making statements based on opinion; back them up with references or personal experience.


Use MathJax to format equations. MathJax reference.


To learn more, see our tips on writing great answers.




draft saved


draft discarded














StackExchange.ready(
function () {
StackExchange.openid.initPostLogin('.new-post-login', 'https%3a%2f%2fmath.stackexchange.com%2fquestions%2f3037830%2fhelp-in-solving-a-problem-posed-in-international-mathematical-olympiad-1983%23new-answer', 'question_page');
}
);

Post as a guest















Required, but never shown





















































Required, but never shown














Required, but never shown












Required, but never shown







Required, but never shown

































Required, but never shown














Required, but never shown












Required, but never shown







Required, but never shown







Popular posts from this blog

Biblatex bibliography style without URLs when DOI exists (in Overleaf with Zotero bibliography)

ComboBox Display Member on multiple fields

Is it possible to collect Nectar points via Trainline?